The phenomenon that causes the position of the Earth's celestial poles to move among the stars called

Answers

Answer 1

The phenomenon that causes the position of the Earth's celestial poles to move among the stars is called precession.

Precession is a slow and gradual wobbling of the Earth's rotational axis caused by the gravitational pull of the Sun and Moon on the Earth's equatorial bulge. This means that over time, the North and South celestial poles appear to move in a circle among the stars. In addition to the precession, the Earth's axial tilt (the angle at which the Earth's North Pole is tilted relative to the plane of the ecliptic) also changes as the precession cycle goes through its 26,000-year period. This causes the position of the celestial poles to move among the stars at a rate of approximately 50 arc seconds per year.

To learn more about stars click here https://brainly.com/question/14354741

#SPJ11


Related Questions

wo ice skaters, paula and ricardo, initially at rest, push off from each other. ricardo weighs more than paula.

Answers

When two ice skaters initially at rest, Paula and Ricardo, push off from each other, the motion they experience is governed by the laws of conservation of momentum. The momentum of a system before and after a collision or interaction remains constant, given that there are no external forces acting on it.

In this case, when Paula and Ricardo push off each other, they both experience equal and opposite forces, according to Newton's Third Law. However, since Ricardo weighs more than Paula, he has a greater mass, which means he has a higher inertia.

This means that he will be less affected by the same force as Paula and will move less than she does.

Thus, when they push off each other, Paula will move more than Ricardo, but the total momentum of the system will remain the same.

This concept is used in many real-world applications, such as rocket propulsion, where the ejection of propellant mass creates a force that propels the rocket forward.

To know more about  ice skaters refer here:

https://brainly.com/question/29475652#

#SPJ11

A bomb, initially at rest, explodes into several pieces.
(a) Is linear momentum of the system (the bomb before the explosion, the pieces after the explosion) conserved?
Yes
No
insufficient information

Answers

The linear momentum of the system the bomb before the explosion, the piece after the explosion is conserved. Therefore, while linear momentum is conserved, other forms of energy are not.

The explosion, the bomb was at rest, so its momentum was zero. After the explosion, the pieces will move in different directions with different velocities, but the sum of their momenta will still be zero. This means that the total momentum of the system is conserved. However, it should be noted that the kinetic energy of the system is not conserved as some of it is lost in the form of heat, sound, and other forms of energy during the explosion. Therefore, while linear momentum is conserved, other forms of energy are not.

learn more about momentum here.

https://brainly.com/question/30677308

#SPJ11

make sure your calculator is in radian mode for this problem, and that you switch it back after this problem. there are two particles (1 and 2) that are moving around in space. the force that particle 2 exerts on 1 is given by: where the parameters have the values: , , . we will consider a time interval that begins at and ends at . impulse from 2 on 1, find the component of the impulse from 2 on 1 between and .

Answers

To find the component of the impulse from particle 2 on particle 1 between t=0 and t=pi/6, we first need to calculate the impulse itself.

The impulse is given by the integral of the force over the time interval, so we have:

J = ∫ F dt (from t=0 to t=pi/6)

Plugging in the given values for the parameters, we get:

J = ∫ (6sin(2t) - 2sin(4t)) dt (from t=0 to t=pi/6)

Evaluating the integral gives us:

J = [ -3cos(2t) + (1/2)cos(4t) ] (from t=0 to t=pi/6)

J = (-3cos(pi/3) + (1/2)cos(pi/2)) - (-3cos(0) + (1/2)cos(0))

J = (-3/2 + 1/2) - (-3 + 1/2)

J = -1

So the impulse from particle 2 on particle 1 between t=0 and t=pi/6 is -1. This means that particle 2 is applying a force to particle 1 in the opposite direction of particle 1's motion during this time interval.

It is important to note that we must ensure our calculator is in radian mode for this problem, and switch it back afterwards to avoid any potential errors in future calculations.

To know more about impulse refer here:

https://brainly.com/question/31213812#

#SPJ11

The Physics of Energy | 1st Edition Chapter 31, Problem 1P Compute the pressure at a depth Z below the surface in a reservoir behind a hydroelectric dam. Compute the work done by a volume of water as it passes from this pressure on one side of a turbine to essentially zero pressure on the other side. Show that this analysis yields the same formula (31.2)[P = e * dV/dt = rho * g * Z * e * Q] for the power output as the energy analysis presented in §31.1.1.

Answers

The analysis using pressure and work yields the same formula for power output as the energy analysis presented in §31.1.1.

To compute the pressure at a depth Z below the surface in a reservoir behind a hydroelectric dam, we can use the formula for hydrostatic pressure: P = rho * g * Z, where rho is the density of water, g is the acceleration due to gravity, and Z is the depth below the surface.To compute the work done by a volume of water as it passes from this pressure on one side of a turbine to essentially zero pressure on the other side, we can use the formula for work: W = P1 * V1 - P2 * V2, where P1 and P2 are the pressures on either side of the turbine, and V1 and V2 are the volumes of water on either side.We can substitute the expression for P1 in terms of Z and simplify the expression to obtain: W = rho * g * Z * e * Q, where e is the efficiency of the turbine and Q is the volume flow rate of water through the turbine.This expression for work is the same as the formula for power output presented in §31.1.1, which is P = e * dV/dt, where dV/dt is the rate of change of volume flow rate with time. By equating the two expressions for work and power output, we obtain the formula for power output in terms of pressure and volume flow rate: P = rho * g * Z * e * Q. Therefore, the analysis using pressure and work yields the same formula for power output as the energy analysis presented in §31.1.1.

For more such question on power

https://brainly.com/question/1634438

#SPJ11

the position vector r of a particle points along the positive direction of the z axis. in what direction is the force producing the torque, if the torque on the particle is (a) zero, (b) in the negative x direction, and (c) in the negative y direction?

Answers

If the position vector r of particle points along the positive direction of the z-axis, the particle is located above the xy-plane. then answers are given below

(a) If the torque on the particle is zero, then the force producing the torque must be perpendicular to the z-axis, i.e., it lies in the xy-plane.
(b) If the torque on the particle is in the negative x-direction, then the force producing the torque must be in the negative y-direction, i.e., it lies in the xy-plane and is perpendicular to the position vector r.
(c) If the torque on the particle is in the negative y-direction, then the force producing the torque must be in the positive x-direction, i.e., it lies in the xy-plane and is perpendicular to both the position vector r and the force producing the torque in part (b).

Learn more about position vector here:

https://brainly.com/question/14552074

#SPJ11

Why is the apparent weight of an object in air greater than its apparent weight when partially or totally immersed in water?The real weight is the weight of the object in a vacuum. The apparent weight is the weight of the object when partially or totally immersed in a fluid e.g. air or water. (And before anyone tries to correct me, a fluid is something that flows; i.e a liquid or a gas.)Apparent weight = weight in a vacuum - upthrust In order to understand this, we need a bit of physics and a bit of maths.
I’ll keep things simple by considering a cube with the upper and lower faces horizontal. You don’t have to, but the maths gets very messy if you consider a complex object … and the result is the same. This is a simple analysis that a Y10 or Y11 student can understand.
The physics we need is that P = F/A; pressure is force divided by area. You can rearrange this formula to give
F = P x A.
The second bit of physics we need is to know that the pressure in a liquid increases with depth. Pressure due to the weight of a liquid of constant density is given by:
P=rhogh
where
P is the pressure,
h is the depth of the liquid,
rho is the density of the liquid, and
g is the acceleration due to gravity.
(Some people might now be getting worried that we are mixing up vectors and scalars willy-nilly. For now, please just take my word that it’s OK.)
WE can combine these two equations to get
F = =rhoghA
We can shift things around a little to make that
F = =rhogAh
and realise that, for a cube, Ah = the volume, V, so it becomes:
F = =rhogV and this is the weight of the fluid displaced.
Now the only problem is to understand which direction this force acts. Well, it acts upwards because the force on the lower face of the cube is greater because of the greater depth. We call this the upthrust.
Since the density of water is greater than the density of air, the upward force is greater. And because of this, the apparent weight is less.
Note, we don’t normally consider the variation of air pressure with height. That’s because the air pressure at the ceiling of a room is pretty much the same as the air pressure at floor level. But the physics is the same. To make life simpler, we consider that the actual weight of an object is equal to its weight in air.
This is an entertaining video that shows what I’m talking about, but without the maths.
6.6K views
View 5 upvotes
Answer requested by Safal Gautam

Answers

The apparent weight of an object in air is greater than its apparent weight when partially or totally immersed in water because of the difference in upthrust, which is the upward force exerted by the fluid on the object.

The real weight of an object is its weight in a vacuum, while the apparent weight is the object's weight when partially or totally immersed in a fluid like air or water.

Apparent weight = real weight - upthrust

To understand this concept, consider a simple cubic object with horizontal upper and lower faces. The pressure in a fluid increases with depth, so the force exerted on the object can be represented by:
F = rhoghA
where F is the force,
P is the pressure,
h is the depth,
rho is the density of the fluid,
g is the acceleration due to gravity, and
A is the area.

Since Ah (the product of area and height) represents the volume (V) of the cube, the equation can be simplified to:
F = rhogV

This force is the weight of the fluid displaced, and it acts upwards due to the greater force on the lower face of the cube because of the greater depth. This upward force is called the upthrust.

The density of water is greater than the density of air, so the upthrust in water is greater than the upthrust in air. As a result, the apparent weight of an object is less when it is partially or totally immersed in water compared to its apparent weight in air.

To know more about "Apparent weight" refer here:

https://brainly.com/question/14323035#

#SPJ11

industrial scrubbers and electrostatic precipitators collect enormous amounts of particulate matter (coal ash) at coal-burning power plants. which of the following best describes an environmental disadvantage of using industrial scrubbers and electrostatic precipitators for pollution abatement?

Answers

One environmental disadvantage of using industrial scrubbers and electrostatic precipitators for pollution abatement is that they generate a large amount of solid waste, which needs to be disposed of safely. The coal ash collected by these devices can contain heavy metals and other pollutants, which pose a risk to human health and the environment if not managed properly.

Disposing of this waste in landfills can lead to contamination of soil and groundwater, while storing it on-site can create the risk of spills and releases. Additionally, the energy required to operate these devices can contribute to greenhouse gas emissions and climate change.

While industrial scrubbers and electrostatic precipitators can effectively collect particulate matter from coal-burning power plants, there are some environmental disadvantages associated with their use.

One major disadvantage is the production of waste materials that must be disposed of. Both types of pollution control systems produce waste materials that contain the collected particulate matter. These waste materials can be hazardous and require special handling and disposal procedures to prevent contamination of soil and water. If not properly disposed of, these waste materials can have negative impacts on the environment.

Overall, while industrial scrubbers and electrostatic precipitators can be effective at controlling particulate matter emissions from coal-burning power plants, there are significant environmental disadvantages that must be carefully considered in their use.

To know more about the industrial scrubbers refer here :

https://brainly.com/question/9603082#

#SPJ11

g a truck with a mass of 1650 kg and moving with a speed of 11.5 m/s rear-ends a 605 kg car stopped at an intersection. the collision is approximately elastic since the car is in neutral, the brakes are off, the metal bumpers line up well and do not get damaged. find the speed of both vehicles after the collision in meters per second. vcar

Answers

The velocity of car during the collision is 12.95m/s and the truck's velocity is 8.41m/s.

Momentum and kinetic energy are both preserved in an elastic collision. These conservation principles may be used to calculate the ultimate velocities of the truck and vehicle.

First, we can use the law of conservation of momentum to find the velocity of the truck after the collision:

[tex]m_{truck} * v_{truck-initial} = m_{truck} * v_{truck-final} + m_{car} * v_{car-final}[/tex]

where

[tex]m_{truck}[/tex] = 1650 kg (mass of the truck)

[tex]v_{truck-initial}[/tex] = 11.5 m/s (initial velocity of the truck)

[tex]m_{car}[/tex] = 605 kg (mass of the car)

[tex]v_{car-final}[/tex] =  the final velocity of the car which is zero, since it is stopped

[tex]v_{truck-initial}[/tex] = the final velocity of the truck

Simplifying the equation and solving for [tex]v_{truck-final}[/tex], we get:

[tex]v_{car-final} = m_{truck} * v_{truck-initial} / m_{truck} + m_{car}[/tex]

[tex]v_{truck-final}[/tex]= (1650 kg * 11.5 m/s)/(1650 kg + 605 kg) = 8.41m/s

Therefore, the velocity of the truck after the collision is 8.41 m/s.

Next, we can use the law of conservation of kinetic energy to find the velocity of the car after the collision:

[tex]1/2 *( m_{truck} * v_{truck-initial} ^{2} ) = (1/2 *m_{truck} * v_{truck-final}^{2} ) + 1/2*( m_{car} * v_{car-final}^{2} )[/tex]

Simplifying the equation and solving for [tex]v_{car-final}[/tex], we get:

[tex]v_{car-final} = \sqrt{(m_{truck} / m_{car}) * v_{truck-initial}^{2} - v_{truck-final}^{2}[/tex]

[tex]v_{truck-final}[/tex] = √((1650 kg/605 kg)*(11.5 m/s)² - (8.41 m/s)²)

= √(2.72 * 61.52)

= √(167.78)

= 12.95m/s

Therefore, the velocity of the car after the collision is 12.95 m/s.

Learn more about Kinetic Energy:

https://brainly.com/question/25959744

#SPJ4

Analyzing the Data:
3. Try to figure out what the data and the results of the investigation mean. Is there a
relationship between the number of paper clips this magnet could attract and the
distance from the magnet the paper clips were placed? What do you think? (2 points)
I
Draw a conclusion:

Answers

According to the data supplied, there is a link between the number of paper clips the magnet could attract and the distance the paper clips were positioned from the magnet.

How to determine objective relationship?

The amount of paper clips attracted reduced as the distance rose. This implies that when one moves away from the magnet, the intensity of the magnetic field weakens.

As a result, the intensity of a magnet's magnetic field is proportional to distance, and the farther an object is from the magnet, the less magnetic force it will experience.

Find out more on magnet here: https://brainly.com/question/14997726

#SPJ1

A wire carries a 12. 55 μA current. How many electrons pass a given point on the wire in 2. 39 s? Round to two decimal places and express your answer in terms of scientific notation, for example: 3. 2.00E+11

Answers

Answer:

Q = N e     where N is number of electrons and Q is total charge

I = Q / t       where I is current and t = sec

I = Q / t = 12.55E-6 Coul / Sec

Q = 12.55E-6 Coul/sec * 2.39 sec = 3.00E-5 Coul    total charge

N = 3.00E-5 coul / 1.60E-19 coul = 1.87E14 electrons

(electronic charge = 1.60E-19 Coul)

the levels of radiation detected by a geiger counter when brought near a sample of radium. the amount of radiation it takes to activate a polyethylene container (turn it radioactive). the amount of radiation to which an airplane passenger is exposed on a transcontinental flight. the total amount of radiation a spacecraft computer chip can withstand before failing because of radiation damage

Answers

Radium generates high radiation levels, while polyethylene resists activation.

When a geiger counter is brought near a sample of radium, it will detect relatively high levels of radiation. Radium is a highly radioactive element, emitting alpha, beta, and gamma radiation.

The geiger counter measures these emissions and provides a reading indicating the intensity of radiation.

The amount of radiation required to activate a polyethylene container, turning it radioactive, is dependent on various factors, such as the thickness and composition of the container.

However, polyethylene is generally considered a poor candidate for activation through radiation exposure, as it is relatively resistant to becoming radioactive.

During a transcontinental flight, an airplane passenger is exposed to cosmic radiation, primarily in the form of high-energy cosmic rays. The exact amount of exposure varies based on factors like altitude, flight duration, and the flight path taken.

However, the level of radiation exposure during a typical transcontinental flight is generally considered low and poses no significant health risks.

The total amount of radiation a spacecraft computer chip can withstand before failing due to radiation damage depends on the chip's design and the radiation-hardening techniques employed.

Specialized chips used in spacecraft are typically designed to withstand higher levels of radiation than commercial chips. They can tolerate radiation doses ranging from several thousand to millions of grays, depending on the specific chip and its protective measures.

For more such questions on Radium, click on:

https://brainly.com/question/27723608

#SPJ11

(a) What is the frequency of the 193nmultraviolet radiation used in laser eye surgery?(b) Assuming the accuracy with which this EM radiation can ablate the cornea is directly proportional to wavelength, how much more accurate can this UV be than the shortest visible wavelength of light?

Answers

The frequency of the 193nm ultraviolet radiation used in laser eye surgery is approximately 1.55 x 10¹⁵ Hz.
The UV radiation used in laser eye surgery is approximately 1.97 times more accurate than the shortest visible wavelength of light.

(a) To calculate the frequency of the 193nm ultraviolet radiation used in laser eye surgery, we can use the formula:
frequency (f) = speed of light (c) / wavelength (λ)
where the speed of light (c) is approximately 3.0 x 10⁸ meters per second (m/s), and the wavelength (λ) is 193nm (or 193 x 10⁻⁹ meters).
So,
f = (3.0 x 10⁸ m/s) / (193 x 10⁻⁹ m)
f ≈ 1.55 x 10¹⁵Hz
The frequency of the 193nm ultraviolet radiation used in laser eye surgery is approximately 1.55 x 10¹⁵ Hz.

(b) To determine how much more accurate the UV radiation is compared to the shortest visible wavelength of EM radiation, we first need to know the shortest visible wavelength. The shortest visible wavelength is around 380nm (violet light).
Next, we can calculate the accuracy ratio by dividing the shortest visible wavelength by the UV wavelength used in laser eye surgery:
accuracy ratio = (380nm) / (193nm)
accuracy ratio ≈ 1.97
The UV radiation used in laser eye surgery is approximately 1.97 times more accurate than the shortest visible wavelength of light.

To learn more about electromagnetic radiation https://brainly.com/question/1408043

#SPJ11

A 5.00-kg sphere is moving at a speed of 4.00 m/s. An identical sphere is at rest. The two spheres collide. The first sphere moves off at a 60.0° angle to the left of its original path. The second sphere moves off in a direction 90.0° to the right of the first sphere’s final path. Assuming no friction, what are the speeds of the two spheres as they separate?

Answers

The final speeds of the spheres are 3.47 m/s and 3.08 m/s.

We can use conservation of momentum to solve this problem since there are no external forces acting on the system.

The initial momentum of the system is:

p_initial = m₁ * v₁ + m₂ * v₂

where m₁ and m₂ are the masses of the spheres, and v₁ and v₂ are their initial velocities (4.00 m/s and 0 m/s, respectively).

After the collision, the momentum of the system is:

p_final = m₁ * v1' + m₂ * v₂'

where v₁' and v₂' are the final velocities of the spheres. We also know that the angle between the first sphere's final path and its initial path is 60 degrees, which means that the angle between the two spheres after the collision is 150 degrees (90 + 60).

Using conservation of momentum, we can set the initial and final momenta equal to each other:

m₁ * v₁ + m₂ * v₂ = m₁ * v₁' + m₂ * v₂'

We can also break down the final velocities into their x and y components using trigonometry. Let's define the angle between the first sphere's final path and the x-axis as theta. Now we can use conservation of momentum to solve for the final velocities:

m₁ * v₁ + m₂ * v₂ = m₁ * v₁' * cos(theta) + m₂ * v₂' * cos(150 degrees)

0 = m₁ * v₁' * sin(theta) + m₂ * v₂' * sin(150 degrees)

Solving the first equation for v₂', we get:

v₂' = (m₁ * v₁ + m₂ * v₂ - m₁ * v₁' * cos(theta)) / (m₂ * cos(150 degrees))

Substituting this expression into the second equation and solving for v₁', we get:

v₁' = (m₂ * sin(150 degrees) * v₁ + m₂ * sin(150 degrees) * v₂ + m₁ * sin(theta) * v₁' - m₁ * sin(theta) * m₂ * v₁ * cos(theta) / cos(150 degrees)) / (m₁ * sin(theta))

Plugging in the given values and solving, we get:

v₁' = 3.47 m/s

v₂' = 3.08 m/s

To know more about friction, here

brainly.com/question/14700266

#SPJ1

when light from the sun hits the atmosphere, the different density of the atmosphere causes the light to bend, or______. group of answer choices reflect refract reabsorb retract

Answers

When light from the sun hits the atmosphere, the different density of the atmosphere causes the light to refract, or bend.

When light travels from one medium to another with a different refractive index, it changes its direction, which is known as refraction. This phenomenon occurs when light from the sun enters the Earth's atmosphere, where the density changes gradually, causing the light to bend. This effect is also responsible for other optical phenomena such as the formation of rainbows and the apparent bending of objects when viewed through a transparent material.

Learn more about refractive index here:

https://brainly.com/question/30761100

#SPJ11

Suppose manufacturers increase the size of compact disks so that they made of the same material and have the same thickness as a current disk but have twice the diameter. By what factor will the moment of inertia increase? A. 2 B. 4 C. 8 D. 16

Answers

The moment of inertia will increase by a factor of 4. Answer: B. 4.

The moment of inertia of a uniform thin disk rotating about its center is given by the formula:

I = [tex](1/2)MR^2[/tex]

where M is the mass of the disk and R is the radius of the disk.

If the diameter of the disk is doubled, then the radius will also double. Therefore, the new moment of inertia will be:

I' =[tex](1/2)M(2R)^2 = 2MR^2[/tex]

The ratio of the new moment of inertia to the original moment of inertia is:

I'/I = [tex](2MR^2) / ((1/2)MR^2) = 4[/tex]

Therefore, the moment of inertia will increase by a factor of 4. Answer: B. 4.

To know more about inertia, visit;

https://brainly.com/question/3268780#

#SPJ11

A Crane does 57,000J of work with a force of 74N to lift a beam. How far can the beam be lifted in meters

Answers

The beam can be lifted at a distance of  770.27 meters.

Work is a physical concept that measures the amount of energy transferred when a force is applied over a distance. In order for work to be done, a force must be applied to an object and the object must move in the direction of the force. Work is typically measured in Joules (J) and is a scalar quantity, meaning it has magnitude but no direction.

To calculate the distance the beam can be lifted, we can use the formula:

work = force x distance x cos(theta)

where work is the amount of work done in Joules, force is the force applied in Newtons, distance is the distance the object is moved in meters, and theta is the angle between the force and the direction of movement (which is assumed to be 0 degrees in this case, since the force is directly upward and the beam is lifted vertically).

Solving for distance, we get:

distance = work / (force x cos(theta))

Plugging in the given values, we get:

distance = 57000 J / (74 N x cos(0)) = 770.27 meters (rounded to two decimal places)

Therefore, there is a 770.27-meter lifting capacity for the beam.

To learn more about Workdone click:

https://brainly.com/question/28172139

#SPJ1

If 5x instead of 10x oculars were used in your microscope with the same objectives, what magnifications would be achieved?

Answers

The magnification is doubled when 10x oculars are used instead of 5x in our microscope with the same objectives.

When multiple lenses are lined together, the overall magnification can be calculated by multiplying the individual magnifications of each lens.

M = M1 × M2 × M3 × ... × Mn

where M is the overall magnification and M1, M2, M3, ..., Mn are the magnifications of the individual lenses.

Let M be the magnification of the objective, then the overall magnification,

when 5x ocular is used,

M1 = M × 5

M1 = 5M

when 10x ocular is used

M2 =  M × 10

M2 = 10M

Therefore, the magnification is doubled when 10x ocular is used instead of 5x in our microscope with the same objectives.

To know more about magnification, click here:

https://brainly.com/question/21370207

#SPJ12

According to the article, how were the gravitational waves generated?

Answers

According to the article, the gravitational waves were generated by the collision of two black holes that were located over a billion light-years away from Earth. This collision caused a massive release of energy in the form of ripples in the fabric of space-time, which is what gravitational waves are.

The black holes were initially orbiting each other at close to the speed of light before they finally merged into a single, more massive black hole. This process caused a massive distortion in space-time that sent gravitational waves radiating outwards in all directions. The waves were detected by the Laser Interferometer Gravitational-Wave Observatory (LIGO) in 2015, marking the first direct observation of gravitational waves in history. This discovery was a major breakthrough in physics and astronomy, as it confirmed the existence of gravitational waves, which were predicted by Einstein's theory of general relativity over a century ago. It also opened up a new window into the study of the universe and its most violent and energetic events.

According to the article, gravitational waves were generated through a powerful cosmic event. This event typically involves the acceleration of massive objects, such as the merging of two black holes or the explosion of a supernova. As these massive objects interact, they cause disturbances in the fabric of spacetime, which leads to the generation of gravitational waves.

These waves then propagate through the universe at the speed of light, carrying information about the events that created them. Advanced detectors, such as LIGO and Virgo, have been designed to measure these tiny ripples in spacetime, enabling scientists to study these events and improve our understanding of the universe.

In summary, the article describes the generation of gravitational waves as a result of the interaction and acceleration of massive objects in the cosmos. These waves carry information about their sources and allow scientists to explore previously unobservable phenomena in the universe.

Learn more about cosmos at : brainly.com/question/3035821

#SPJ11

in uniform circular motion, which of the following are constant: speed, velocity, angular velocity, centripetal acceleration, magnitude of the net force?

Answers

In a uniform circular motion, the speed and magnitude of the net force are constant, while the velocity, angular velocity, and centripetal acceleration are not constant.

Speed refers to the rate at which an object is moving, and in a uniform circular motion, the object moves at a constant speed around a fixed point. The magnitude of the net force is also constant because the force required to maintain the circular motion is always the same.

However, the velocity is not constant because the direction of the object's motion is constantly changing. The angular velocity, which refers to the rate at which the object rotates around the fixed point, is also not constant because the object is moving at a constant speed but the distance it travels in one rotation changes as it moves in a circular path.

Lastly, the centripetal acceleration, which is the acceleration towards the center of the circle, is also not constant because it depends on the speed and radius of the circular path.

Overall, understanding the constants and variables in uniform circular motion is important in understanding the mechanics of circular motion and its applications in physics.

To know more about uniform circular motion refer here:

https://brainly.com/question/2285236#

#SPJ11

The MPC for a country will likely be measured as less than 1. 0. T True F False

Answers

The statement is True, The MPC for a country will likely be measured as less than 1.

MPC in physics stands for "Multipurpose Ceramic". However, it's unclear what specific context you are referring to as MPC could stand for many different things in physics, depending on the field and application. For example, in particle physics, MPC could stand for "Minimum Projected Calorimeter", which is a type of calorimeter used to measure the energy of particles.

In astrophysics, MPC could refer to "Minor Planet Center", which is an organization responsible for collecting and disseminating information about minor planets, comets, and natural satellites. In materials science, MPC could refer to "Metal-Plastic Composite", which is a type of material made by combining metal and plastic components. In optics, MPC could refer to "Micro-structured Polymer Composite", which is a material used for making diffractive optical elements.

To learn more about MPC visit here:

brainly.com/question/29645795

#SPJ4

a -3.0 c charge and a 2.0 c charge are placed 0.60 m apart. part a (1 points) what is the magnitude of the electric dipole moment of this charge distribution?

Answers

The magnitude of the electric dipole moment of this charge distribution is 1.2 C⋅m.

What is the magnitude of the electric dipole moment of a charge distribution?

The electric dipole moment of a charge distribution is defined as the product of the magnitude of the charge and the distance between the charges multiplied by a unit vector pointing from the negative charge to the positive charge.

In this case, we have a -3.0 C charge and a 2.0 C charge placed 0.60 m apart. Let's assume that the -3.0 C charge is located at the origin and the 2.0 C charge is located at a point (0.60, 0).

The magnitude of the electric dipole moment can be calculated as:

p =q * d

where q is the magnitude of the charge and d is the distance between the charges.

In this case, q = 2.0C and d = 0.60m

Therefore:

p =(2.0C) * (0.60m)p = 1.2C.m

So the magnitude of the electric dipole moment of this charge distribution is 1.2 C⋅m.

Learn more about Electric dipole

brainly.com/question/31500545

#SPJ11

the buoyant force acts upon the . group of answer choices center of mass center of gravity center of volume center of gyration

Answers

The buoyant force acts upon the center of volume of an object immersed in a fluid. This is the point at which the volume of the object is balanced in all directions by the surrounding fluid.

However, it is important to note that the center of volume may not necessarily coincide with the object's center of mass, center of gravity, or center of gyration. These points are determined by other factors such as the distribution of mass or the shape of the object.

1. An object submerged in a fluid experiences a buoyant force.
2. This buoyant force is equal to the weight of the fluid displaced by the object.
3. The buoyant force acts upward, opposing the object's weight.
4. The point at which this force is applied is the center of volume, which is the geometric center of the displaced fluid.

So, the buoyant force acts upon the center of volume.

To know more about buoyant force, refer

https://brainly.com/question/11884584

#SPJ11

PART OF WRITTEN EXAMINATION:
High conductivity
A) reduces the ability to support current flow
B) indicates an ability to support current flow
C) resistances the ability to support current flow

Answers

High conductivity B) indicates an ability to support current flow because the material offers minimal resistance. This property is essential in various applications, such as in the construction of electrical circuits and components, where efficient current flow is crucial to achieving optimal performance


High conductivity refers to a material's ability to efficiently conduct an electric current. Materials with high conductivity typically have low resistances, which means they do not hinder the flow of electric current. In contrast, materials with low conductivity have high resistances and obstruct the flow of electric current, making it more difficult for the current to pass through them.
When a material has high conductivity, it can easily support the flow of electric current because there is minimal resistance. This means that electrons can easily move through the material without losing energy or generating excessive heat. Examples of materials with high conductivity include metals such as copper, silver, and gold.
On the other hand, materials with low conductivity or high resistances, such as insulators like rubber, plastic, and glass, make it difficult for the current to flow. This is because these materials have a structure that does not allow electrons to move freely, leading to a build-up of energy and increased heat.

To learn more about current flow, refer:-

https://brainly.com/question/2264542

#SPJ11

The voltage required to stop an electron that was ejected from the cathode in a photoelectric effect experiment is 0. 65 V (also called the stopping voltage).

What is the maximum kinetic energy of the ejected electron?

Note: 1 J = 6. 242×1018 ev

Answers

Answer:

Stopping voltage (V) = 0.65 V

1 electronvolt (eV) = 1.602 × 10^-19 joules (J)

Maximum kinetic energy (K) of the ejected electron = ?

K can be calculated using the formula: K = eV

First, convert V to joules using the conversion factor 1 eV = 1.602 × 10^-19 J

V in joules = 0.65 V x 1.602 × 10^-19 J/eV = 1.043 × 10^-19 J

Therefore, K = eV = 0.65 eV x 1.602 × 10^-19 J/eV = 1.0443 × 10^-19 J

How does the kinetic energy of cart 2 change, if cart 1 has the initial energy K1,i = 120J ?
Express your answer to two significant digits and include the appropriate units. Enter positive value if the energy increases and negative value if the energy decreases.
As a result of an elastic collision between carts 1 and 2, the kinetic energy of cart 1 increases four times.

Answers

The kinetic energy of cart 2 increases by 360 J to two significant digits.

The kinetic energy of cart 2 will increase by a factor of 4 and will have a final energy of K2,f = 480 J. This is because kinetic energy is conserved in an elastic collision, meaning that the total kinetic energy before the collision (K1,i + K2,i) is equal to the total kinetic energy after the collision (K1,f + K2,f).

Since K1,f = 4K1,i = 480 J,

we can rearrange the equation to solve for K2,

f, which is equal to K2,f = K1,i + K2,i - K1,f = 120 J + K2,i - 480 J = -360 J + K2,i

. Therefore, K2,f = 480 J. The kinetic energy of cart 2 increases by 360 J.
In an elastic collision, the total kinetic energy is conserved. If the initial kinetic energy of cart 1 is K1,i = 120 J and its kinetic energy increase four times after the collision, the final kinetic energy of cart 1 becomes K1,f = 4 * K1,i = 480 J.

Since the total kinetic energy is conserved, the change in kinetic energy of cart 2, ΔK2, can be found using the equation:

ΔK2 = K1,f - K1,i = 480 J - 120 J = 360 J

Therefore, the kinetic energy of cart 2 increases by 360 J to two significant digits.

learn more about kinetic energy

https://brainly.com/question/26472013

#SPJ11

a '29er' mounbtain bike wheel has a diameter of 29.0 in . what is the moment of inertia of this wheel (expressed in standard units)? the rim and tire have a combined mass of 0.850 kg . remember that 1in

Answers

The moment of inertia of the wheel is 0.0564 kg [tex]m^{2}[/tex]

To calculate the moment of inertia of the 29er mountain bike wheel, we need to know the mass distribution of the wheel. Let's assume that the mass of the wheel is concentrated in the rim and tire, which is a reasonable approximation.

The moment of inertia of a hoop (or a circular rim) is given by the formula:

I = \frac{1}{2} m r^{2}[/tex]

where I is the moment of inertia, m is the mass of the hoop, and r is the radius of the hoop. Since we know the diameter of the wheel is 29.0 inches, the radius is 14.5 inches (which is equal to 0.3683 meters, using the conversion factor you provided).

The mass of the rim and tire is given as 0.850 kg. To convert this mass to the mass of the hoop, we need to subtract the mass of the hub and spokes, which we do not have information about. Let's assume that the mass of the hub and spokes is negligible compared to the mass of the rim and tire. In this case, the mass of the hoop is equal to the mass of the rim and tire.

Therefore, the moment of inertia of the 29er mountain bike wheel is:

I = \frac{1}{2} m r^{2}[/tex]

= (1/2) * 0.850 kg * (0.3683 m)^2[tex]= \frac{1}{2} *0.850 kg * (0.3683)^{2} m\\= 0.0564kg m^{2}[/tex]

So the moment of inertia of the wheel is 0.0564 kg [tex]m^{2}[/tex], expressed in standard units.

Learn more about the moment of inertia:

https://brainly.com/question/14460640

#SPJ11

help please!! I'm pretty sure the answer is E.

Answers

Answer:

The answer is indeed E, 4K1.

Explanation:

When the block is compressed a distance x from equilibrium, the spring exerts a restoring force on the block given by Hooke's law:

F = -kx

where k is the spring constant. The negative sign indicates that the force is in the opposite direction to the displacement.

As the block is released, this restoring force accelerates the block to the right. At any point during the motion, the total mechanical energy (kinetic plus potential) of the system is conserved. Initially, all the energy is potential energy stored in the compressed spring. At the point when the block separates from the spring, all the potential energy has been converted into kinetic energy. Therefore, we have:

K = (1/2)mv1^2 = (1/2)kx^2

where v1 is the speed of the block when it separates from the spring.

When the block is compressed a distance 2x, the spring exerts a restoring force given by:

F = -2kx

This force is twice as large as the force when the block was compressed a distance x. Therefore, the block will experience twice the acceleration and reach twice the speed when it separates from the spring. The kinetic energy of the block at this point is given by:

K' = (1/2)mv2^2 = (1/2)k(2x)^2 = 4kx^2

where v2 is the speed of the block when it separates from the spring after being compressed a distance 2x.

So the ratio of the kinetic energies when the block is released from compressions of distance x and 2x respectively is:

K'/K = 4kx^2 / (1/2)kx^2 = 8

Therefore, the kinetic energy of the block when it separates from the spring after being compressed a distance 2x is 8 times the kinetic energy when it is compressed a distance x, i.e., K' = 8K. So the answer is E, 4K1

7. A submarine is 30m below sea water of density 1g/cm³. if the atmospheric pressure at the place is equivalent to 760mmHg. Find the total pressure acting on the submarine (Take density of mercury =13600kg/m³) ​

Answers

The total pressure acting on the submarine is equal to 2967.19 mmHg.

To find pressure at a depth of 30 m under the sea surface by using the formula:

P = ρgh

P = pressure,

ρ = density of the liquid

g = acceleration due to gravity

h = depth

According to question

density of seawater = 1g/cm³, which is equivalent to 1000 kg/m³

1g/cm³ = 1000 kg/m³, and

h is equal to  30 m,

We can find the pressure on the submarine by using:

Pressure = ρgh

Pressure = 1000 kg/m³ × 9.81 m/s² × 30 m

Pressure = 294300 Pa

To calculate the total pressure to act upon the submarine, add the atmospheric pressure to the pressure due to the seawater.

According to question atmospheric pressure is 760mmHg, which is equal to 101325 Pa (1mmHg = 133.322 Pa), the total pressure on the submarine can be obtained as:

Total pressure is equal to atmospheric pressure + pressure due to seawater

P = 101325 Pa + 294300 Pa

P = 395625 Pa

To change this pressure into units of mmHg, use the information that 1 Pa = 0.0075 mmHg

Total P in mmHg = 395625 Pa × 0.0075 mmHg/Pa

So, total pressure in mmHg is 2967.19 mmHg.

Learn more about pressure, here:

https://brainly.com/question/31342351

#SPJ1

a pendulm on plant x where the value of g in unknown oscillates with a perod of 2 s. what is the period of theis pendulm if its mass is doubled

Answers

The period of a pendulum is dependent on the length of the pendulum and the acceleration due to gravity (g). Since the value of g on plant X is unknown, we cannot determine the period of the pendulum. However, we can determine how the period would change if the mass of the pendulum is doubled.

According to the formula for the period of a pendulum, T = 2π√(L/g), where T is the period, L is the length of the pendulum, and g is the acceleration due to gravity. Since we are doubling the mass of the pendulum, it means that the force acting on the pendulum will also be doubled. Therefore, the equation can be rewritten as T = 2π√(L/2g).
Simplifying this expression, we can see that the period of the pendulum will increase by a factor of √2, which is approximately 1.41. Therefore, if the original period of the pendulum was 2 seconds, the new period of the pendulum would be 2 x √2 = 2.83 seconds.

learn more about mass here

https://brainly.com/question/13492374

#SPJ11

two oscillating systems: spring-mass and simple pendulum undergo shm with an identical period t. if the mass in each system is doubled which of the following is true about the new period?

Answers

The new period denoted as T', of both the spring-mass and simple pendulum systems after doubling the mass in each system will remain unchanged and be equal to the original period T.

The period of a simple harmonic motion (SHM) is determined by the properties of the system, such as the mass and the restoring force. In the case of a spring-mass system, the period is given by the equation T = 2π√(m/k), where m is the mass of the object attached to the spring and k is the spring constant.

In the case of a simple pendulum, the period is given by the equation T = 2π√(L/g), where L is the length of the pendulum and g is the acceleration due to gravity.

When the mass in each system is doubled, the mass term in the equations gets multiplied by 2. However, the square root of the mass term remains unchanged, as the square root of 2 is still the same value. Therefore, the new period T' of both systems will remain the same as the original period T, as the effect of doubling the mass is canceled out by the square root operation in the period equation.

This result holds true for idealized scenarios where other factors such as air resistance, damping, and non-linearities are negligible. In real-world scenarios, these factors may affect the actual period of the systems.

To know more about simple harmonic motion (SHM) refer here:

https://brainly.com/question/30404816#

#SPJ11

Other Questions
For the following reaction, if nh3 is used up at a rate of 0. 30mmin, what is the rate of formation of h2? 2nh3n2 3h2 What is a baseline product backlog? When homeostasis is disturbed by increasing CO2 level & decreasing pH and O2 levels are cardioacceleratory centers stimulated or inhibited? The makers of Aspaway brand aspirin want to be sure that their tablets contain the right amount of active ingredient (acetylsalicylic acid). So they inspect a random sample of 30 tablets from a batch in production. When the production process is working properly, Aspaway tablets have an average of = 320 milligrams (mg) of active ingredient. The amount of active ingredient in the 30 selected tablets has a mean of 319 mg and a standard deviation of 3 mg. We want to perform a test at the a= 0.05 significance level of H: = 320 H: 320 where = the mean amount of active ingredient (in mg) in all Aspaway brand aspirin tablets. AlCl4- + H+ = AlCl3 + HCl. (True or False) A 2.02-kg particle has a velocity (2.03 2.96 ) m/s, and a 2.94-kg particle has a velocity (0.91 + 5.94 ) m/s.(a) Find the velocity of the center of mass. + m/s(b) Find the total momentum of the system. + kg m/s Which two of the following states are east of the Mississippi River?(Select all that apply.)ArkansasMissouriKentuckyTennessee A random variable X has a mean of 120 and a standard deviation of 15. A random variable Y has a mean of 100 and a standard deviation of 9. If X and Y are independent, approximately what is the standard deviation of X - Y Which of the following online behaviors can lead to negative legal consequences? (Choose all that apply)--------------------DoxingCyberbullyingArguing or talking backHarassment How does your culture shape your health? Give an example of the cultural beliefs that determine how family treats illness. In a normal distribution, what percentage of the data falls within 2 standarddeviations of the mean?A. 99.7%B. 68%C. 34%D. 95% what is the molarity of a solution that contains 32.00g sodium chlorid and 275g of water (i will give brainliest) What two factors enabled Songhai to become a powerful trading center? who composed Organ Fugue in G minor? a parallelogram has a base of 6 centimeters and a height of 10 centimeters. this parallelogram has the same area as a triangle with a height of 5 centimeters. what is the measure, in centi,of the base of the triangle? Half reaction for conversion of sulphite to sulfate What major arteries and veins deliver and drain blood to and from the head and neck? What are the lasting lessons that we can learn from studying this event? Support your response with evidence. 70 yo insulin-dependent diabetic M presents with episodes of confusion, dizziness, palpitation, diaphoresis, and weakness. What the diagnostic? Q: A good answer to someone offering you ADHD medicine is: